LSAT and Law School Admissions Forum

Get expert LSAT preparation and law school admissions advice from PowerScore Test Preparation.

 Administrator
PowerScore Staff
  • PowerScore Staff
  • Posts: 8917
  • Joined: Feb 02, 2011
|
#35076
Complete Question Explanation

Justify the Conclusion—SN. The correct answer choice is (B)

Your task in this Justify the Conclusion question is to select the answer choice containing
information that, if added to the argument, would show the activist’s conclusion is valid.

..... Premise: ..... any member of the city council ought either to vote against the propose or
..... ..... ..... ..... abstain
..... ..... ..... ..... ..... ..... ..... ..... ..... vote against the proposal

..... ..... ..... ..... city council member ..... :arrow: ..... or

..... ..... ..... ..... ..... ..... ..... ..... ..... ..... abstain


..... Premise: ..... if all members abstain, the matter will be decided by the city’s voters

..... ..... ..... all members abstain ..... :arrow: ..... matter decided by city’s voters

..... Conclusion: ..... thus, at least one member of the city council should vote against the proposal

..... ..... ..... ..... ..... ..... ..... all members abstain

The correct answer in this Justify the Conclusion question will provide information sufficient
to show that at least one member of the council should vote against the proposal. Saying that at
least one member votes against the proposal has the same logical effect as saying not all members
abstain, the necessary condition of the contrapositive to the conditional relationship in the second
sentence. Thus, your prephrase is that the correct answer will provide the sufficient condition for the
contrapositive, that the matter should not be decided by the city’s voters.

The incorrect answer choices will not justify the conclusion, meaning they may merely support the
conclusion, have no effect on the conclusion, or weaken it.

Answer choice (A): While incorrect, this answer choice is attractive, because it appears from the
activist’s argument that the activist opposes the proposal, and would conclude it would be a bad thing
for the city’s voters to decide in favor of it. However, the issue in the argument was not whether the
proposal is passed, but whether the city’s voters should decide the matter.

Answer choice (B): This is the correct answer choice. This choice is correct, because it provides
the sufficient condition for the contrapositive to the conditional statement in the second sentence,
showing the conclusion to be valid.

Answer choice (C): This information has no effect on the conclusion. To say that no members of
the city council will vote in favor of the proposal is the same as saying the members will either vote
against the proposal or abstain.

Answer choice (D): This choice is the Mistaken Negation of the conditional relationship in —the
second sentence:


..... ..... ..... ..... all members abstain ..... :arrow: ..... matter decided by city’s voters

Answer choice (E): This choice has no effect on the conclusion. Instead, it provides a view regarding
what should occur if the situation described in the conclusion occurs.
 Arindom
  • Posts: 76
  • Joined: Apr 11, 2016
|
#23377
Hi,

I had some trouble on this question. Could you check my diagramming?
MCC - Member of city council
A - abstain
VAP - vote against proposal
MDCV - matter will be decided by city voters

Premise 1: MCC ---- VAP or A
Premise 2 : A -----MDCV
Conclusion: MCC ---- VAP

Is this diagramming correct? If so, then where can I plug in/get to "the proposal should bot be decided by the city's voters"?

For these types of questions, is it advisable to diagram? What is the best way to approach these types of questions?

Thanks,
- Arindom
 Emily Haney-Caron
PowerScore Staff
  • PowerScore Staff
  • Posts: 577
  • Joined: Jan 12, 2012
|
#23458
Hi Arindom,

Generally, your diagramming looks good. The one thing I would say is you want to make sure you're distinguishing between ANY MCC (first sentence) and at least one MCC.

Basically, what is missing here is something explaining why we do not want MDCV, or why it should be avoided. That will connect the premises to the conclusion.

For this one, I would definitely diagram, but I wouldn't be overly rigid about it - you want to make sure you're getting the big picture, rather than just applying everything to the diagram, because the big picture is what you're looking for with the correct answer choice here. Does that make sense?
 LilyRose
  • Posts: 19
  • Joined: Mar 13, 2016
|
#26019
I have a question about this, but I did not diagram.

I had my answers narrowed down to two contenders, A and B. As far as I can tell, neither one of these is better than the other; I used the ANT and both seemed to weaken the argument? Tell me please, did I do it correctly:

A. Even if all of the embers of the city council abstain in the vote on the proposal, the city's voters will not necessarily decide in favor of the proposal.
(? I struggle to negate conditionals, but as far as I can tell, Nikki's 'even if' method is the most reliable)
B. The proposal should be decided by the city's voters.

B does directly weaken the argument, but A seems to weaken as well by leaving open the possibility that the city's voters will decide in favor.

As far as I can tell, the activist thinks that the city voters will vote in favor, and she thinks that the proposal should not pass, so therefore the vote should not go to the city voters. I guess B gets there much more directly. Is this right?
 Robert Carroll
PowerScore Staff
  • PowerScore Staff
  • Posts: 1787
  • Joined: Dec 06, 2013
|
#26021
Lily,

Be careful! The word "assumed" does not make this an Assumption question. In an Assumption question, the wording of the question will indicate that the author already assumed the answer or that the argument depends on the assumption in the answer choice, or some such wording. In fact, that's why the Assumption Negation Technique works in Assumption questions - the correct answer choice was an assumption needed by the argument, so the logical opposite of that assumption should weaken the argument.

We don't have that kind of question type here, though. The question does not say the assumption is already made, but rather that, if assumed, the correct answer choice will cause the conclusion of the argument to follow logically. In other words, the argument works perfectly if the correct answer choice is added to the argument. That makes this a Justify question.

Thus, your thinking has to be adjusted to the requirements of a Justify. The correct answer choice must prove the conclusion (when added to the premises that already exist). There is a gap in the argument. It is not established that having the matter decided by the city's voters is a bad thing, yet the author is arguing that the city council needs to take steps to prevent the voters from deciding. Answer choice (A) still does not establish why it's a bad thing, a thing that needs to be prevented, to let the voters decide. Only answer choice (B) makes the appropriate link by establishing that letting the voters decide should be avoided.

Robert Carroll
 LilyRose
  • Posts: 19
  • Joined: Mar 13, 2016
|
#26023
D'oh!.... it is a Justify, not an Assumption.... and now I understand why ANT does not work. I see now why A is wrong.

Thank you very much!
 deck1134
  • Posts: 160
  • Joined: Jun 11, 2018
|
#49358
Is this a question that we should diagram on? Or should we approach it more abstractly?

I am not sure what the best way is.

Thanks.
 hassan66
  • Posts: 51
  • Joined: Jul 19, 2018
|
#59363
Hi,

I am still having trouble understanding why C is incorrect. Would this be the answer if it were a necessary assumption? The stimulus says that the city council members "ought" to either abstain or vote against the bill, it doesn't say that they actually will do so. So if you say that at least one member of the council should vote against the bill, you're assuming that no one will vote in favor of the bill. Was my mistake in approaching the problem as a necessary assumption question and not as a justify question?

Thank you!
 Robert Carroll
PowerScore Staff
  • PowerScore Staff
  • Posts: 1787
  • Joined: Dec 06, 2013
|
#60973
hassan,

The argument is entirely about what should happen. Information about what does happen is irrelevant. This statement of yours: "So if you say that at least one member of the council should vote against the bill, you're assuming that no one will vote in favor of the bill." is incorrect. I could say that the members should do something, even if no one ever does. Nothing about their actual actions necessarily impacts what they should do.

For this reason, answer choice (C) would still be wrong for an Assumption question.

Robert Carroll
 ericau02
  • Posts: 73
  • Joined: Feb 19, 2019
|
#64329
I see how this is right but I am confused a bit. I thought that ,maybe C would be right in saying that no members will vote in favor meaning that they don't want to vote in favor thats why they are making one city member council abstain.

secondly, for B I for some reason wrote down that it was incorrect because the stimulus never really focused on what the city voters should do but more on what the council members should do. I know this is incorrect but I am hoping you can help me try to to understand why my logic is off. I feel as if I am mixing question types and applying something i saw from one question into another.
If that makes sense?

Get the most out of your LSAT Prep Plus subscription.

Analyze and track your performance with our Testing and Analytics Package.